PrepTest 10 (February 1994), Section B, Question #24 Forum

Prepare for the LSAT or discuss it with others in this forum.
Post Reply
User avatar
LSAT_Padawan

New
Posts: 45
Joined: Thu Feb 18, 2010 10:17 pm

PrepTest 10 (February 1994), Section B, Question #24

Post by LSAT_Padawan » Mon Mar 01, 2010 1:08 am

Must Be True question where the author states (paraphrasing) the rate of inflation exceeds the rate of return on the most profitable investment available, the difference between those two rates will be the percentage by which, at a minimum, the value of any investment will decline. If in such a circumstance the value of a particular investment declines by more than that percentage, it must be true that _______________.

Why is it (C) "the investment in question is less profitable than the most profitable investment available?" Why not (B)" the investment in question is becoming less profitable" or (D) "the rate of return on the most profitable investment available has declined?"

Thank you for your elaboration.

r6_philly

Diamond
Posts: 10751
Joined: Sat Dec 19, 2009 4:32 pm

Re: PrepTest 10 (February 1994), Section B, Question #24

Post by r6_philly » Mon Mar 01, 2010 1:28 am

scenario:

Inflation: 10%

Investment A:
9% rate of return (most profitable)
difference: -1%

Investment B:
5% rate of return (not most profitable)
difference: -5%

So the stimulus describes Most profitable (Inv. A) is declining by 1% (-1%). then it goes on to ask: if a particular investment (Inv. B) is declining more than that rate (-1% vs -5%), it ___

(b) states the investment in question is becoming less profitable: This is true. But remember the stem compared its rate to the -1% rate (the best rate), the credited answer need to address that rate.
(c) is the better answer. Because the decline rate of -5% is worse than the -1% of the most profitable investment, then this investment is not the most profitable.
(d) the rate of return DID NOT decline. The rate of return is the 9% that is said in the stem. It is still "the most profitable" - the investment with the highest rate of return.

I think you need to know that the rate of return will always be positive (well in this case). The stock is losing value beause the inflation has made the dollar value worse less even though it is earning a high rate of return at the same time. (this is not said in the question, you just have to understand how investments work)

User avatar
LSAT_Padawan

New
Posts: 45
Joined: Thu Feb 18, 2010 10:17 pm

Re: PrepTest 10 (February 1994), Section B, Question #24

Post by LSAT_Padawan » Mon Mar 01, 2010 1:38 am

Thank you r6. Your illustration was quite helpful.

r6_philly

Diamond
Posts: 10751
Joined: Sat Dec 19, 2009 4:32 pm

Re: PrepTest 10 (February 1994), Section B, Question #24

Post by r6_philly » Mon Mar 01, 2010 1:42 am

No problem. I will lurk here until my score comes out in a week so ask away!

Actually I was off on my explanation of (b). The investment is not becoming less profitable, it is just less worthwhile :)

Want to continue reading?

Register now to search topics and post comments!

Absolutely FREE!


Post Reply

Return to “LSAT Prep and Discussion Forum”